3
$\begingroup$

Set $G_\eta:=\{(x,y)\in \mathbb{R}^2|-\eta<x<\eta, 0<y<1\}$. If $u_\eta\geq 0$ is a sequence of subharmonic functions defined on $G_\eta$ such that $$ \int_{G_\eta}|u_\eta|^2dx\wedge dy\leq C\eta, $$ where $C$ is some constant, then can we show that $u_\eta$ is uniformly bounded on the line $0\times [\frac{1}{3},\frac{2}{3}]\subset \mathbb{R}^2$?

$\endgroup$

2 Answers 2

1
$\begingroup$

This is incorrect as stated: it just does not scale correctly. Let $v(z)$ be the harmonic function on the unit square $|x|<1,|y|<1,$ which is zero on the vertical sides and $1$ on the horizontal sides. Then we define the subharmonic function $u$ in your rectangle by setting $u(z)=v(z/\eta)$ when $|x|<\eta,\; |y|<\eta$ and zero otherwise. Then $u(0)=v(0)$ is a positive constant, while $\| u\|_2=\| v\|_2\eta$, so $$\int_{G_\eta}u^2\approx c\eta^2.$$

This example suggests the correct inequality: $$u(x)\leq C\| u\|_2/\eta,$$ with an absolute constant $C$. This is a simple consequence of the average property of subharmonic functions. Let $x$ be a point on your interval $(1/3,3/2)$. Let $B$ be the disk of radius $\eta$ centered at $x$. Then the average property says that $$u(x)\leq \int_B u\; dm/(\pi\eta^2),$$ where $dm$ is the Lebesgue area element and $\pi\eta^2$ is the area of the disk. Applying the Schwarz (Cauchy-Bunyakovski-...) inequality, we obtain $$\left(\int_B u\; dm\right)^2\leq\pi\eta^2\int_Bu^2dm\leq\pi\eta^2\| u\|_2^2,$$ so $$\int_Bu\; dm\leq\sqrt{\pi}\eta\| u\|_2,$$ and combining with the first inequality we obtain the result with constant $\sqrt{\pi}$.

$\endgroup$
2
  • $\begingroup$ This doesn't work since the $L^2$ norm of $u$ is bounded by $\sqrt{\eta}$ not $\eta$. $\endgroup$
    – foliations
    Mar 30, 2016 at 22:50
  • $\begingroup$ @foliations: I edited. It is of course incorrect as stated. $\endgroup$ Mar 30, 2016 at 23:09
0
$\begingroup$

No. A counterexample is (essentially) given by $$ u(z) = -\log |z+i\delta|, \quad\quad z=x+iy, y\ge 0 . $$ This function is harmonic and positive near $z=0$, and $|u(x+iy)|\le |u(x)|\in L^2(-1/2,1/2)$, so $$ \int_{-1/2}^{1/2} dx\int_0^{2\eta} dy\, u^2 \lesssim \eta , $$ as required. However, $|u(i\eta)|$ is not bounded as $\delta,\eta\to 0+$.

By rotating and (slightly) rescaling this, we obtain a counterexample in your setting.

$\endgroup$
5
  • $\begingroup$ An easy way to see that his inequality cannot be correct is just by scaling: if you multiply all his functions on constants, sup norms multiply on the same constants, but the integral of the square on the square of the constant. $\endgroup$ Mar 31, 2016 at 2:30
  • $\begingroup$ @AlexandreEremenko: Yes, but I think what I do is natural, too: solve the Dirchlet problem on a thin rectangle with a boundary value that is large only on a small portion of the boundary and check if the area average of $u^2$ can stay bounded. $\endgroup$ Mar 31, 2016 at 2:35
  • $\begingroup$ Sure. BTW the description in your comment better describes the example in my answer:-) $\endgroup$ Mar 31, 2016 at 2:39
  • $\begingroup$ @AlexandreEremenko: Well, I tried it with a Poisson kernel first (which fits my description better), but that's too singular after squaring, so switched to a logarithm then... :) $\endgroup$ Mar 31, 2016 at 15:18
  • $\begingroup$ thanks a lot for all of your answers. I know the point in this question. $\endgroup$ Mar 31, 2016 at 19:20

Your Answer

By clicking “Post Your Answer”, you agree to our terms of service and acknowledge you have read our privacy policy.

Not the answer you're looking for? Browse other questions tagged or ask your own question.